Integrating legendre polynomials with weighting function

Click For Summary
The integral in question involves the product of Legendre polynomials weighted by a sine function, specifically from 0 to π and its equivalent form from -1 to 1. The user notes the absence of a straightforward analytic solution in standard references like Gradshteyn and Ryzhik. They propose a complex method involving Taylor series expansion and triple integrals of Legendre polynomials, but find it too convoluted for practical use. The challenge lies in integrating the product of Legendre functions outside the typical intervals. The discussion seeks alternative approaches to evaluate this integral effectively.
fantispug
Messages
101
Reaction score
0

Homework Statement


I recently came across this integral while doing a problem in electromagnetism (I'm not sure if there exists a nice analytic answer):

\int_{0}^{\pi}P_m(\cos(t))P_n(\cos(t)) \sin^2(t) = \int_{-1}^{1}P_m(x) P_n(x) \sqrt{1-x^2},

Homework Equations



P_m(x) is the m^th Legendre Polynomial.

The Attempt at a Solution



There are lots of close integrals in Gradshteyn and Ryzhik 7.1-7.2 but nothing close enough for me to use.

One way to evaluate it would be to expand the square root as a Taylor series, and then change basis to re-expand it as a series of Legendre polynomials, then use tricks involving triple integrals of Legendre polynomials (such as those in Arfken and Weber 12.9). However this is incredibly messy and I can't see how I could get an analytic expression from it.

Can anyone think of a nice way of approaching this integral? I can't use for example differentiation under the integral because I don't know how to integrate the product of Legendre functions on intervals other than [-1,1] (and [0,1]).
 
Physics news on Phys.org
Never mind...
 
Question: A clock's minute hand has length 4 and its hour hand has length 3. What is the distance between the tips at the moment when it is increasing most rapidly?(Putnam Exam Question) Answer: Making assumption that both the hands moves at constant angular velocities, the answer is ## \sqrt{7} .## But don't you think this assumption is somewhat doubtful and wrong?

Similar threads

  • · Replies 1 ·
Replies
1
Views
2K
  • · Replies 1 ·
Replies
1
Views
3K
  • · Replies 5 ·
Replies
5
Views
2K
  • · Replies 2 ·
Replies
2
Views
3K
  • · Replies 2 ·
Replies
2
Views
3K
  • · Replies 1 ·
Replies
1
Views
880
Replies
2
Views
2K
  • · Replies 3 ·
Replies
3
Views
2K
Replies
1
Views
2K
Replies
1
Views
2K